Constructing a Sequence to Show the Existence of a Limit

Click For Summary
A sequence (v_n) in a non-empty, upper bounded subset V of R can be constructed to show that the limit is sup V. If sup V is in V, the sequence can be defined as v_0 = v_1 = ... = sup V. If sup V is not in V, a recursive construction is suggested where each term v_{i+1} is chosen from the interval (v_i, sup V) to ensure it approaches sup V. The sequence must be defined such that each term is greater than the previous one, ensuring convergence to sup V. The discussion emphasizes the importance of selecting terms in a way that guarantees the limit of the sequence is indeed sup V.
Anoonumos
Messages
16
Reaction score
0
Hi,

Homework Statement


V is a non-empty, upper bounded subset of R. Show that a sequence (v_n)_{n \geq 0} in V exists such that: 1)v_0 \leq v_1 \leq ... and 2) the limit of the sequence is sup V. (Hint: use a recursive construction)


Homework Equations





The Attempt at a Solution


V is non-empty and upper bounded so sup V exists.
Suppose sup V \in V Construct the sequence: sup V = v0 = v1...
And I know how to go from there.

I'm having problems with the case: sup V is not in V.
I was thinking of constructing the sequence:
v_0 \in V
(v_n, supV) \subset (v_{n-1}, supV)

But I'm not sure if this is right or how to proof that the limit of this sequence is sup V.
Any ideas?
 
Physics news on Phys.org
Let v_0 be any member of V. We can do this because V is non-empty.

The recursion step: for any v_i, (sup V+ v_i)/2< sup(V) and so is not an upper bound on V/. There must exist some v_{i+1} in V such that (v_i+ sup(V))/2< v_{i+1}. That cuts the distance from the previous term to sup(V) in half on each step.
 
Thanks!
 
Sorry for the double post, but I came across another problem when writing it down. v(i+1) doesn't have to exist right? And there doesn't have to be a smallest v(i+1). So how would one formulate the sequence?

v_{i+1} \in [v_1, sup V) perhaps?

Or is it enough to state v(i+1) exists and v(i+1) ? vi and v(i+1) is in V, to formulate a sequence?
 
Last edited:
No, v_{i+1} by this method must exist: Since in this case we are assuming that sup(V) is not in V itself, our (v_i+ sup(V))/2 is NOT sup(V) and so is not an upper bound for V. There must exist at least one member of V larger than (v_i+ sup(V))/2. I said nothing about v_{i+1} being the smallest such member of V- choose any of them.

But it is not sufficient just to say that v_{i+1} is some number in V larger than v_i. That sequence would not necessarily converge to sup(V).
 
I understand. I have one final question.
Nevermind, thanks :)
 
Last edited:
Question: A clock's minute hand has length 4 and its hour hand has length 3. What is the distance between the tips at the moment when it is increasing most rapidly?(Putnam Exam Question) Answer: Making assumption that both the hands moves at constant angular velocities, the answer is ## \sqrt{7} .## But don't you think this assumption is somewhat doubtful and wrong?

Similar threads

  • · Replies 13 ·
Replies
13
Views
2K
  • · Replies 14 ·
Replies
14
Views
3K
  • · Replies 1 ·
Replies
1
Views
2K
Replies
6
Views
3K
  • · Replies 16 ·
Replies
16
Views
3K
  • · Replies 28 ·
Replies
28
Views
3K
  • · Replies 6 ·
Replies
6
Views
2K
  • · Replies 9 ·
Replies
9
Views
2K
  • · Replies 11 ·
Replies
11
Views
3K
Replies
3
Views
2K